Kathleen Jackson
Quiz by , created more than 1 year ago

Module 1 EoT Exam Revision

1489
6
0
Kathleen Jackson
Created by Kathleen Jackson almost 6 years ago
Close

2808NRS Human Pathophysiology and Pharmacology 2 (Module 1)

Question 1 of 130

1

A 56-year-old male is diagnosed with coronary heart disease. Which of the following modifiable risk factors would most likely influence development of this disease?

Select one of the following:

  • cigarette smoking.

  • his age.

  • the geographic location of home.

  • his sex.

Explanation

Question 2 of 130

1

In general, atherosclerosis is triggered by:

Select one of the following:

  • injured neutrophils.

  • deposited adipose cells.

  • macrophages that engulf LDL.

  • lipid-laden mast cells.

Explanation

Question 3 of 130

1

Plaques consisting of lipids, cells, fibrin, and debris, often with attached thrombi that can obstruct arteries and arterioles, are referred to as:

Select one of the following:

  • Atheromas

  • Sarcomas

  • Fibromas

  • Prothrombi

Explanation

Question 4 of 130

1

The most common cause of myocardial ischaemia is:

Select one of the following:

  • venous emboli.

  • atherosclerosis.

  • vasospasm.

  • arterial emboli from heart valve vegetation.

Explanation

Question 5 of 130

1

A 32-year-old female presents with lower leg pain and redness. An ultrasound reveals venous thrombus. Which of the following would be most likely to have caused her condition?

Select one of the following:

  • atherosclerosis.

  • heart valve damage.

  • bacterial infection.

  • vascular injury and inflammation.

Explanation

Question 6 of 130

1

A 75-year-old obese female presents to her GP complaining of oedema in the lower extremities. Physical exam reveals that she has varicose veins. Which of the following is a most likely cause?

Select one of the following:

  • ischaemia.

  • extreme exercise.

  • long periods of standing.

  • dyslipidemia.

Explanation

Question 7 of 130

1

A 28-year-old female presents with severe chest pain and shortness of breath. She is diagnosed with pulmonary embolism, which most likely originated from the:

Select one of the following:

  • deep veins of the leg.

  • left ventricle.

  • systemic arteries.

  • superficial veins of the body.

Explanation

Question 8 of 130

1

A 52-year-old male presents with pooling of blood in the veins of the lower extremities and oedema. He is predisposed to:

Select one of the following:

  • deep vein thrombosis.

  • skin hyperpigmentation.

  • stasis ulcers.

  • all answers are correct.

Explanation

Question 9 of 130

1

A 65-year-old female presents to the ED complaining of difficulty swallowing and shortness of breath. A CT scan would most likely reveal an aneurysm in the:

Select one of the following:

  • cerebral vessels.

  • illiac arteries.

  • inferior vena cava.

  • thoracic aorta.

Explanation

Question 10 of 130

1

The build-up of fatty plaques within the wall of arteries is:

Select one of the following:

  • Ischaemic heart disease

  • Peripheral vascular disease

  • Atherosclerosis

  • Hypertension

Explanation

Question 11 of 130

1

Which of the following types of drugs is NOT typically used to treat hypertension?

Select one of the following:

  • Thiazide diuretics

  • Nitrates

  • ACE inhibitors

  • Calcium channel blockers

Explanation

Question 12 of 130

1

Hypertension that is idiopathic is known as:

Select one of the following:

  • Angina

  • Secondary

  • Essential

  • Tertiary

Explanation

Question 13 of 130

1

Risk factors for atherosclerosis include all the following except:

Select one of the following:

  • Low levels of LDL blood serum

  • Hypertension

  • Smoking

  • Advanced age

Explanation

Question 14 of 130

1

A stationary blood clot within an artery is a:

Select one of the following:

  • Myocardial infarction

  • Embolus

  • Stroke

  • Thrombus

Explanation

Question 15 of 130

1

A 62-year-old male presents to his GP complaining of chest pain at rest and on exertion. He does not have a history of coronary artery disease and reports that the pain often occurs at night. He is most likely experiencing which type of angina?

Select one of the following:

  • Exertional

  • Stable

  • Variant

  • Unstable

Explanation

Question 16 of 130

1

A 53-year-old male presents with recurrent chest pain on exertion. He is diagnosed with angina pectoris. The pain he experiences occurs when:

Select one of the following:

  • the vagus nerve is stimulated.

  • myocardial stretch has exceeded the upper limits.

  • the myocardial oxygen supply has fallen below demand.

  • cardiac output has fallen below normal levels.

Explanation

Question 17 of 130

1

A 51-year-old male is at his health clinic for an annual physical exam. After walking from the car to the clinic, he developed a substernal pain. He also reported discomfort in his left shoulder and his jaw, lasting two to three minutes and then subsiding with rest. He indicates that this has occurred frequently over the past few months with similar exertion. He is most likely to be experiencing:

Select one of the following:

  • stable angina

  • unstable angina

  • variant angina

  • myocardial infarction

Explanation

Question 18 of 130

1

A 52-year-old female is diagnosed with coronary heart disease. She would be expected primarily to suffer from myocardial:

Select one of the following:

  • hypertrophy.

  • ischaemia.

  • necrosis.

  • dilation.

Explanation

Question 19 of 130

1

Cardiac cells can withstand ischaemic conditions for _____ before irreversible cell injury occurs.

Select one of the following:

  • 30 minutes

  • 60 minutes

  • 1 minute

  • 20 minutes

Explanation

Question 20 of 130

1

A 49-year-old male presents to his GP complaining of chest pain. An ECG reveals ST elevation. He is diagnosed with myocardial ischaemia. Which of the following interventions would be most beneficial?

Select one of the following:

  • decrease in ventricular volume with diuretic

  • increase cardiac contractility

  • increase heart rate

  • increase myocardial oxygen supply

Explanation

Question 21 of 130

1

A 55-year-old male died of myocardial infarction. Autopsy would most likely reveal:

Select one of the following:

  • platelet aggregation within an atherosclerotic coronary artery.

  • decreased ventricular diastolic filling time.

  • embolisation of plaque from the aorta.

  • smooth muscle dysplasia in the coronary artery.

Explanation

Question 22 of 130

1

A 75-year-old male presents with severe chest pain. Lab tests in hospital reveal elevated levels of creatine kinase and lactic dehydrogenase. These elevated levels indicate probable:

Select one of the following:

  • Unstable angina

  • Hypertension

  • Stable angina

  • Myocardial infarction

Explanation

Question 23 of 130

1

A 60-year-old female had a myocardial infarction. She was brought to hospital 30 minutes later. She survived but now has impaired ventricular function because:

Select one of the following:

  • there was a temporary alteration in electrolyte balance.

  • the resulting ischaemia led to hypoxic injury and myocardial cell death.

  • there was too much stress on the heart.

  • the cells became hypertrophic.

Explanation

Question 24 of 130

1

Low-density lipoproteins (LDL):

Select one of the following:

  • transport cholesterol from cells to the liver for excretion.

  • contain only small amounts of cholesterol.

  • promote atheroma development.

  • are associated with low intake of saturated fats.

Explanation

Question 25 of 130

1

Which of the following patients will likely experience difficulty in maintaining lipoprotein synthesis resulting in elevated LDL levels?

Select one of the following:

  • A 35-year-old patient with a history of hepatitis C and B with end-stage liver disease

  • A 44-year-old female admitted for hysterectomy due to cervical cancer with metastasis

  • A 27-year-old patient with pancreatitis related to alcohol abuse

  • A 55-year-old male admitted for exacerbation of chronic obstructive pulmonary disease (COPD)

Explanation

Question 26 of 130

1

Hypertension can be classified into the following three major categories:

Select one of the following:

  • Active, passive, fatal

  • Primary, secondary, tertiary

  • Subacute, acute, chronic

  • Essential, secondary, malignant

Explanation

Question 27 of 130

1

Where do atherosclerotic plaques occur most often?

Select one of the following:

  • in large, abdominal veins

  • at points of arterial vessel bifurcation

  • in myocardial capillaries

  • all answers are correct

Explanation

Question 28 of 130

1

Calcium-channel blocking drugs are effective in:

Select one of the following:

  • decreasing all types of cardiac arrhythmias.

  • reducing the risk of blood clotting.

  • decreasing the attraction of cholesterol into lipid plaques.

  • reducing cardiac and smooth muscle contractions.

Explanation

Question 29 of 130

1

Which of the following confirms the presence of a myocardial infarction?

Select one of the following:

  • The presence of elevated serum cardiac enzymes and triglycerides

  • Serum cardiac enzymes markers released from necrotic cells and ECG changes

  • Leukocytosis and elevated C-reactive protein

  • A full description of the pain, including the sequence of development

Explanation

Question 30 of 130

1

High levels of which of the following lipoproteins can be beneficial?

Select one of the following:

  • IDL

  • LDL

  • HDL

  • VLDL

Explanation

Question 31 of 130

1

A 52-year-old male is diagnosed with primary hypertension. He could be treated with a drug that acts by which of the following mechanisms?

Select one of the following:

  • calcium channel agonist

  • diuretic

  • angiotensin ll agonist

  • beta-adrenergic agonist

Explanation

Question 32 of 130

1

Claudication describes, or is related to:

Select one of the following:

  • Painful cramps in skeletal muscles due to peripheral vascular disease

  • Painful upper limbs due to hypertension

  • Chest pain

  • All answers are correct

Explanation

Question 33 of 130

1

The basic pathophysiology of myocardial infarction is best described as:

Select one of the following:

  • total obstruction of a coronary artery, which causes myocardial necrosis.

  • irregular heart rate and force, reducing blood supply to coronary arteries.

  • temporary vasospasm that occurs in a coronary artery.

  • cardiac output that is insufficient to meet the needs of the heart and body.

Explanation

Question 34 of 130

1

Factors that may precipitate an angina attack include all of the following EXCEPT:

Select one of the following:

  • eating a large meal.

  • engaging in an angry argument.

  • running upstairs on a hot day.

  • taking a nap.

Explanation

Question 35 of 130

1

Typical early signs or symptoms of myocardial infarction include:

Select one of the following:

  • persistent chest pain radiating to the left arm, pallor, and rapid, weak pulse.

  • brief, substernal pain radiating to the right arm, with labored breathing.

  • bradycardia, increased blood pressure, and severe dyspnoea.

  • flushed face, rapid respirations, left-side weakness, and numbness.

Explanation

Question 36 of 130

1

Four patients were admitted to the emergency department with severe chest pain. All were given preliminary treatment with aspirin, morphine, and nitrates and were monitored by ECG. Which patient most likely experienced myocardial infarction?

Select one of the following:

  • A 67-year-old female whose pain started at 2 AM while she was asleep and responded to nitrates; the ECG showed arrhythmias and ST-segment elevation; cardiac markers remained stable.

  • An 80-year-old woman whose pain started at 6 AM shortly after awakening and was not relieved by nitrates or rest; the ECG showed ST-segment elevation; levels of cardiac markers subsequently rose.

  • A 33-year-old male whose pain started at 7 AM during moderate exercise and was relieved by nitrates; ECG was normal; cardiac markers remained stable.

  • A 61-year-old man whose pain started at 9 AM during a short walk and responded to nitrates, but not to rest; ECG and cardiac markers remained stable, but anginal pattern worsened.

Explanation

Question 37 of 130

1

Cigarette smoking is a risk factor in coronary artery disease because smoking:

Select one of the following:

  • promotes platelet adhesion.

  • increases serum HDL levels.

  • decreases serum lipid levels.

  • reduces vasoconstriction and peripheral resistance.

Explanation

Question 38 of 130

1

When comparing angina with myocardial infarction (MI), which statement is true?

Select one of the following:

  • Pain is more severe and lasts longer with angina than with MI.

  • Both angina and MI cause tissue necrosis.

  • Angina pain is relieved by rest and intake of nitroglycerin; the pain of MI is not.

  • Angina often occurs at rest; MI occurs during a stressful time.

Explanation

Question 39 of 130

1

Which of the following hypertensive individuals is most likely to have his or her high blood pressure diagnosed as secondary rather than essential?

Select one of the following:

  • A 40-year-old smoker who eats excessive amounts of salt and saturated fats

  • An African American man who leads a sedentary lifestyle

  • A 69-year-old woman who is an executive in a large corporation

  • A 51-year-old male who has been diagnosed with glomerulonephritis

Explanation

Question 40 of 130

1

A 54-year-old man with a long-standing diagnosis of essential hypertension is meeting with his physician. The patient's physician would anticipate that which of the following phenomena is most likely occurring?

Select one of the following:

  • The conversion of angiotensin I to angiotensin II in his lungs causes increases in blood pressure and sodium reabsorption.

  • The patient's juxtaglomerular cells are releasing aldosterone as a result of sympathetic stimulation.

  • ADH is exerting an effect on his chemoreceptors and baroreceptors resulting in vasoconstriction.

  • Adrenalin from his adrenal gland is initiating the renin -angiotensin- aldosterone system.

Explanation

Question 41 of 130

1

What condition refers to thrombotic events associated with venous wall inflammation, and occurs in any part of the body?

Select one of the following:

  • phlebothrombosis

  • deep vein thrombosis

  • varicose veins

  • thrombophlebitis

Explanation

Question 42 of 130

1

Which vessel/s is/are particularly prone to aneurysm?

Select one of the following:

  • aorta

  • superior vena cava

  • small peripheral arteries of the legs

  • inferior vena cava

Explanation

Question 43 of 130

1

A drug taken in small doses on a continuing basis to reduce platelet adhesion is:

Select one of the following:

  • Aspirin

  • Streptokinase

  • Heparin

  • Ibuprofen

Explanation

Question 44 of 130

1

A venous disorder in which a thrombus forms spontaneously in a vein without prior inflammation is referred to as:

Select one of the following:

  • Venous stenosis

  • Thrombophlebitis

  • Fibrothrombosis

  • Phlebothrombosis

Explanation

Question 45 of 130

1

A general term for all types of arterial changes, but most often applied to loss of elasticity of arteries and arterioles, is:

Select one of the following:

  • Vascular stenosis

  • Atherosclerosis

  • Arteriosclerosis

  • Osteoporosis

Explanation

Question 46 of 130

1

A modifiable factor that increases the risk for atherosclerosis is:

Select one of the following:

  • leading a sedentary lifestyle.

  • being female and older than 40 years of age.

  • familial hypercholesterolemia.

  • excluding saturated fats from the diet.

Explanation

Question 47 of 130

1

A coronary artery disease that occurs when the blood flow to the myocardial cells is interrupted for an extended period of time, followed by necrosis, is referred to as:

Select one of the following:

  • Angina pectoris

  • Atherosclerosis

  • Myocardial infarction

  • Dysrhythmias

Explanation

Question 48 of 130

1

Which antihypertensive drug group should be avoided when a person has a pre-existing respiratory condition like asthma?

Select one of the following:

  • Calcium channel antagonists.

  • Diuretics.

  • β-blockers.

  • ACE inhibitors.

Explanation

Question 49 of 130

1

Adverse effects such as an unproductive cough sometimes occur with which category of medicine frequently used for treatment of hypertension?

Select one of the following:

  • Calcium channel blockers

  • Diuretics

  • β-blockers

  • ACE inhibitors

Explanation

Question 50 of 130

1

Factors that may lead to the formation of varicose veins include:

Select one of the following:

  • Trauma, lying in bed for prolonged periods, and liver failure

  • Pregnancy, vitamin deficiencies, and mitral valve defects

  • Vein valve damage, wearing tight clothing, and crossing legs

  • Diet, exercise, and obesity

Explanation

Question 51 of 130

1

How do ACE inhibitors work?

Select one of the following:

  • They block Angiotensinogen production in the lungs

  • They block the enzyme cyclooxygenase

  • They block the formation of the enzyme renin by the kidney

  • They block the conversion of Angiotension 1 to Angiotensin 2

Explanation

Question 52 of 130

1

A 13-year-old boy has had a sore throat for at least a week and has been vomiting for 2 days. His glands are swollen, and he moves stiffly because his joints hurt. Throat cultures show infection with group A streptococci. This child is at high risk for?

Select one of the following:

  • mitral valve stenosis.

  • vasculitis.

  • myocarditis.

  • infective endocarditis.

Explanation

Question 53 of 130

1

A client has been diagnosed with mitral valve stenosis following his recovery from rheumatic fever. Which of the following teaching points would be most accurate to convey to the client?

Select one of the following:

  • Your heart's mitral valve isn't closing properly so blood is flowing backward in your heart and eventually into your lungs.

  • The valve between your left ventricle and left atria is infected and isn't allowing enough blood through.

  • The normal tissue that makes up the valve between the right sides of your heart has stiffened.

  • Your mitral valve isn't opening up enough for blood to flow into the part of your heart that sends blood into circulation.

Explanation

Question 54 of 130

1

On a routine physical exam visit, the physician mentions that he hears a new murmur. The patient gets worried and asks, “What does this mean?” The physician responds;

Select one of the following:

  • One of your heart valves is not opening properly. We need to do an echocardiogram to see which valve is having problem.

  • This may make you a little more fatigued than usual. Let me know if you start getting dizzy or light-headed.

  • This could be caused by an infection. Have you been feeling well the past few weeks?

  • It would be caused by stress. Let's keep our eye on it and see if it goes away with your next visit.

Explanation

Question 55 of 130

1

A 34-year-old man who is an intravenous drug user has presented to the emergency department with malaise, abdominal pain, and lethargy. The health care team wants to rule out endocarditis as a diagnosis. Staff of the department would most realistically anticipate which of the following sets of diagnostics?

Select one of the following:

  • CT of the heart, chest x-ray, and ECG

  • ECG, blood pressure, and stress test

  • Echocardiogram, blood cultures, and temperature

  • Cardiac catheterization, chest x-ray, electrolyte measurement, and white cell count

Explanation

Question 56 of 130

1

A physician has ordered the measurement of a cardiac patient's electrolyte levels as part of the client's morning blood work. Which of the following statements best captures the importance of potassium in the normal electrical function of the patient's heart?

Select one of the following:

  • Potassium catalyzes the metabolism of ATP, producing the gradient that results in electrical stimulation.

  • Potassium is central to establishing and maintaining the resting membrane potential of cardiac muscle cells.

  • The reciprocal movement of one potassium ion for one sodium ion across the cell membrane results in the production of an action potential.

  • The impermeability of cardiac cell membranes to potassium allows for action potentials achieved by the flow of sodium ions.

Explanation

Question 57 of 130

1

The initial medical management for a symptomatic patient with obstructive hypertrophic cardiomyopathy (HCM) would be administering a medication to block the effects of catecholamines. The nurse will anticipate administering which of the following medications?

Select one of the following:

  • Propranolol, a beta-adrenergic blocker

  • Lanoxin, an inotropic

  • Lisinopril, an ACE inhibitor

  • Lasix, a diuretic

Explanation

Question 58 of 130

1

A 63-year-old male client has been diagnosed with a bundle branch block. How will this client's care team most likely expect his condition to be expressed diagnostically?

Select one of the following:

  • His ECG will show a flattened P wave as a result of impaired atrial depolarization.

  • His ECG will show an inordinately wide QRS complex because impulses are bypassing the normal conduction tissue.

  • Conduction from the Purkinje fibers to the bundle branches is compromised by inadequate conduction.

  • His AV node will be performing the primary pacemaker role due to inadequacy of the SA node.

Explanation

Question 59 of 130

1

You are looking after a 61-year-old male client in the hospital who has presented with a new onset of atrial fibrillation. Which of the following courses of treatment would you most likely expect the attending physician to initiate?

Select one of the following:

  • Diuretics, total bed rest, and cardioversion if necessary

  • Anticoagulants and beta-blockers to control rate

  • Antihypertensives and constant cardiac monitoring in a high acuity unit

  • Immediate cardioversion followed by surgery to correct the atrial defect

Explanation

Question 60 of 130

1

The amount of blood pumped by one ventricle in one minute, is called the

Select one of the following:

  • stroke volume.

  • cardiac output.

  • ejection fraction.

  • end-diastolic volume.

Explanation

Question 61 of 130

1

If someone's heart has a stroke volume of 70 ml and a heart rate of 90 beat/minute, the cardiac output would be

Select one of the following:

  • 70 ml/min

  • 6.3 L/min

  • 1.28 ml/min

  • 0.77 L/min

Explanation

Question 62 of 130

1

Any mechanism that increases heart rate is said to have a positive ____ effect.

Select one of the following:

  • chronotropic

  • inotropic

  • cholinergic

  • feedback

Explanation

Question 63 of 130

1

Stroke volume is regulated by all of the following except

Select one of the following:

  • end-diastolic volume

  • peripheral resistance

  • cardiac output

  • contractility

Explanation

Question 64 of 130

1

The preload acting on a ventricle is equivalent to that chamber's

Select one of the following:

  • end-diastolic volume

  • stroke volume

  • contractility

  • ejection fraction

Explanation

Question 65 of 130

1

The afterload imposed on a ventricle refers to

Select one of the following:

  • the amount of blood added to a ventricle by atrial systole

  • the total peripheral resistance opposing the ejection of blood

  • its end-systolic volume, the blood left after contraction is complete

  • the ejection fraction, or percentage of EDV ejected by ventricular systole

Explanation

Question 66 of 130

1

The Frank-Starling law of the heart describes the proportional relationship between

Select one of the following:

  • stroke volume and end-diastolic volume

  • stroke volume and cardiac output

  • stroke volume is and total peripheral resistance

  • the left and right ventricles

Explanation

Question 67 of 130

1

A positive inotropic agent is something that

Select one of the following:

  • decreases the contractility of myocardial fibers

  • increases the contractility of myocardial fibers

  • reduces the heart rate in positive feedback loop

  • increases the heart rate in positive feedback loop

Explanation

Question 68 of 130

1

A nurse is administering morning medications to a number of patients on a medical unit. Which of the following medication regimens is most suggestive that the patient has a diagnosis of heart failure?

Select one of the following:

  • Anticoagulant, antihypertensive, calcium supplement

  • Beta-blocker, potassium supplement, anticoagulant

  • Diuretic, ACE inhibitor, beta-blocker

  • Antihypertensive, diuretic, antiplatelet aggregator

Explanation

Question 69 of 130

1

A nurse will be providing care for a female patient who has a diagnosis of heart failure that has been characterized as being primarily right sided. Which of the following statements best describes the presentation that the nurse should anticipate? The client

Select one of the following:

  • has cyanotic lips and extremities, low urine output, and low blood pressure.

  • has pitting edema to the ankles and feet bilaterally, decreased activity tolerance, and occasional upper right quadrant pain.

  • complains of dyspnoea and has adventitious breath sounds on auscultation (listening).

  • has a distended bladder, facial edema, and nighttime difficulty breathing.

Explanation

Question 70 of 130

1

A 3-year-old child with right-sided heart failure has been admitted for worsening of his condition. Which of the following assessments would be considered one of the earliest signs of systemic venous congestion in this toddler?

Select one of the following:

  • Breathlessness with activity

  • Increased urine output

  • Enlargement of the liver

  • Excessive crying

Explanation

Question 71 of 130

1

The nurse working in the ICU knows that chronic elevation of left ventricular end-diastolic pressure will result in the patient displaying which of the following clinical manifestations?

Select one of the following:

  • Chest pain and intermittent ventricular tachycardia

  • Dyspnoea and crackles in bilateral lung bases

  • Petechia and spontaneous bleeding

  • Muscle cramping and cyanosis in the feet

Explanation

Question 72 of 130

1

Which one of the following is not a pathophysiological change associated with heart failure?

Select one of the following:

  • Decreased angiotensin II production.

  • Decreased myocardial contractility.

  • Increased myocardial oxygen demand.

  • Cardiac remodelling.

Explanation

Question 73 of 130

1

Pulmonary symptoms, such as dyspnoea and cough, common to left heart failure are a result of:

Select one of the following:

  • pulmonary vascular congestion.

  • hypoxaemia.

  • inflammatory pulmonary oedema.

  • bronchoconstriction.

Explanation

Question 74 of 130

1

Adverse effects such as an unproductive cough and taste disturbances may occur from which category of medicine frequently used for treatment of heart failure?

Select one of the following:

  • Aldosterone antagonists.

  • Cardiac glycosides.

  • ACE inhibitors.

  • Diuretics.

Explanation

Question 75 of 130

1

Right heart failure will likely cause:

Select one of the following:

  • non-pitting oedema in the arms, resulting from decreased arterial pressure

  • pitting oedema in the lower legs, resulting from increased venous pressure

  • pulmonary oedema from increased left ventricular filling

  • all answers are correct

Explanation

Question 76 of 130

1

A 65-year-old male is diagnosed with chronic pulmonary disease and elevated pulmonary vascular resistance. Which of the following heart failures generally results from this condition?

Select one of the following:

  • low-output failure

  • right heart failure

  • high-output failure

  • left heart failure

Explanation

Question 77 of 130

1

Excess preload can be reduced by:

Select one of the following:

  • use of antidiuretics

  • use of diuretics

  • increasing volume intake, that is, drinking more fluid

  • all answers are correct

Explanation

Question 78 of 130

1

In terms of cardiac pathology, the greatest danger of untreated rheumatic fever is:

Select one of the following:

  • myocardial hypertrophy

  • damage to heart valves

  • atherosclerosis

  • acute right-side heart failure

Explanation

Question 79 of 130

1

An incompetent mitral valve would cause:

Select one of the following:

  • decreased pressure in the left atrium.

  • decreased output from the left ventricle.

  • hypertrophy of the right ventricle.

  • increased blood to remain in the right atrium.

Explanation

Question 80 of 130

1

A very rapid heart rate reduces cardiac output because:

Select one of the following:

  • conduction through the AV node is impaired.

  • ventricular filling is reduced.

  • venous return is increased.

  • ventricular fibrillation develops immediately.

Explanation

Question 81 of 130

1

Pericarditis causes a reduction in cardiac output as a result of which of the following?

Select one of the following:

  • Delays in the conduction system, interfering with cardiac rhythm

  • Excess fluid in the pericardial cavity, which decreases ventricular filling

  • Weak myocardial contractions due to friction rub

  • Incompetent valves, which allow regurgitation of blood

Explanation

Question 82 of 130

1

A patient, who is experiencing some angina associated with tachycardia, has been placed on verapamil, a calcium channel blocker. Knowing the mechanism of action of this medication, you should assess this patient for which of the following adverse reactions?

Select one of the following:

  • Ventricular tachycardia

  • Bradycardia

  • Increased cardiac output

  • Sudden asystole

Explanation

Question 83 of 130

1

Which of the following drugs improves cardiac efficiency by slowing the heart rate and increasing the force of cardiac contractions?

Select one of the following:

  • Epinephrine

  • Nifedipine

  • Digoxin

  • Furosemide

Explanation

Question 84 of 130

1

Heart block, in which a conduction delay at the AV node results in intermittent missed ventricular contractions, is called:

Select one of the following:

  • total heart block.

  • first-degree block.

  • second-degree block.

  • bundle-branch block.

Explanation

Question 85 of 130

1

Atrial fibrillation may:

Select one of the following:

  • increase the risk of thromboembolism

  • induce an irregular heart rate

  • increase turbulence within the atrial chamber

  • all answers are correct

Explanation

Question 86 of 130

1

The current optimal drug therapy in the management of heart failure is:

Select one of the following:

  • a non-selective β-blocker.

  • the dopamine agonist, dobutamine.

  • the cardiac glycoside, digoxin.

  • an ACE inhibitor and a diuretic.

Explanation

Question 87 of 130

1

Which one of the following would you not expect to be administered to a patient suffering heart failure?

Select one of the following:

  • diuretic

  • digoxin

  • beta-agonist

  • ACE inhibitor

Explanation

Question 88 of 130

1

A 67-year-old female has chronic gastrointestinal bleeding. The primary cause of her anaemia is:

Select one of the following:

  • bone marrow failure

  • folate deficiency

  • vitamin B12 deficiency

  • iron deficiency

Explanation

Question 89 of 130

1

Maternal-fetal blood incompatibility may exist in which of the following conditions?

Select one of the following:

  • The mother is Rh-negative and the fetus is Rh-positive.

  • The mother has type AB blood and the fetus has type B blood.

  • The mother has type A blood and the fetus has type O blood.

  • The mother is Rh-positive and the fetus is Rh-negative.

Explanation

Question 90 of 130

1

A 5-year-old male is diagnosed with normocytic-normochromic anaemia. Which of the following anaemias falls into this category?

Select one of the following:

  • thalassaemia

  • haemolytic anaemia

  • pernicious anaemia

  • iron deficiency anaemia

Explanation

Question 91 of 130

1

A 35-year-old female is diagnosed with vitamin B12 deficiency anaemia. A decrease in which of the following is the most likely cause?

Select one of the following:

  • transferin

  • intrinsic factor

  • gastric enzymes

  • ferritin

Explanation

Question 92 of 130

1

Erythrocytes that are _____ contain an abnormally low concentration of haemoglobin.

Select one of the following:

  • macrocytic

  • hypochromic

  • microcytic

  • hyperchromic

Explanation

Question 93 of 130

1

Which of the following is typically not associated with anaemia?

Select one of the following:

  • increased haemolysis

  • lack of intrinsic factor

  • high dietary iron intake

  • disruption of haemoglobin chains

Explanation

Question 94 of 130

1

Those who live at high altitudes, or who receive extra erythropoietin, are likely to suffer:

Select one of the following:

  • primary polycythaemia

  • haemolytic disease

  • secondary polycythaemia

  • sickle cell anaemia

Explanation

Question 95 of 130

1

Anaemia due to inherited mutations that reduce the production of either alpha or beta haemoglobin chains is known as:

Select one of the following:

  • haemolytic anaemia

  • thalassaemia

  • pernicious anaemia

  • all answers are correct

Explanation

Question 96 of 130

1

Which blood disorder results from an autoimmune attack on parietal cells of the stomach wall?

Select one of the following:

  • haemolytic disease of the newborn

  • pernicious anaemia

  • microcytic anaemia

  • thalassaemia

Explanation

Question 97 of 130

1

What medical term is used to describe a marked reduction in platelets?

Select one of the following:

  • haemorrhoids

  • haemophilia

  • thrombocytopenia

  • polycythaemia

Explanation

Question 98 of 130

1

Multiple opportunistic infections develop with acute leukemia primarily because:

Select one of the following:

  • many circulating leukocytes are immature

  • severe anemia interferes with the immune response

  • the number of white blood cells is decreased

  • decreased appetite and nutritional intake reduce natural defences

Explanation

Question 99 of 130

1

The cause of leukaemia is unknown, but risk factors include ...

Select one of the following:

  • Exposure to high amounts of radiation

  • Genetic disorders such as Down's syndrome

  • Exposure to carcinogens such as benzene

  • All answers are correct

Explanation

Question 100 of 130

1

Why is excessive bleeding a common occurrence with acute leukaemia?

Select one of the following:

  • Deficit of calcium ions

  • Dysfunctional thrombocytes

  • Impaired production of prothrombin and fibrinogen

  • Decreased platelets

Explanation

Question 101 of 130

1

A 5-year-old female is diagnosed with acute leukaemia. She will most likely be treated with:

Select one of the following:

  • bone marrow transplant

  • immunotherapy

  • localised radiation therapy

  • chemotherapy

Explanation

Question 102 of 130

1

The Reed-Sternberg cell is diagnostic for:

Select one of the following:

  • chronic myeloblastic leukemia.

  • multiple myeloma

  • non-Hodgkin’s lymphoma

  • Hodgkin’s lymphoma

Explanation

Question 103 of 130

1

Which of the following applies to the leukaemia’s?

Select one of the following:

  • Exposure to chemicals is not considered a predisposing factor.

  • Lymphoid tissue produces abnormal leukocytes.

  • Chronic leukemias are more common in older people.

  • All answers are correct.

Explanation

Question 104 of 130

1

Predisposing factors to leukemia commonly include:

Select one of the following:

  • exposure to radiation.

  • certain fungal and protozoal infections.

  • familial tendency.

  • cigarette smoking.

Explanation

Question 105 of 130

1

What is the primary treatment for the leukemias?

Select one of the following:

  • Chemotherapy

  • Radiation

  • Surgery

  • Immunotherapy

Explanation

Question 106 of 130

1

Multiple myeloma is a malignant tumor involving:

Select one of the following:

  • bone cells.

  • lymph nodes.

  • plasma cells.

  • granulocytes.

Explanation

Question 107 of 130

1

In cases of polycythemia vera, blood pressure is most likely elevated as a result of:

Select one of the following:

  • increased renin and aldosterone secretions.

  • increased blood volume/viscosity

  • congested spleen and bone marrow.

  • frequent infarcts in the coronary circulation

Explanation

Question 108 of 130

1

Vitamin K is required by the liver to synthesize:

Select one of the following:

  • bilirubin

  • amino acids

  • prothrombin

  • heparin

Explanation

Question 109 of 130

1

_____________________ would result from a reduced number of RBCs in the blood?

Select one of the following:

  • Decreased haematocrit

  • Increased haemoglobin count

  • Decreased osmotic pressure of the blood

  • Increased risk of haemostasis

Explanation

Question 110 of 130

1

Haemolytic disease of the newborn is due to the presence of ________ and ________.

Select one of the following:

  • Rh+ foetal blood, Anti-Rh antibodies in maternal blood

  • Rh+ foetal blood, Rh+ maternal blood

  • Rh- foetal blood, Rh+ maternal blood

  • Anti-Rh antibodies in foetal blood, lack of antibodies in maternal blood

Explanation

Question 111 of 130

1

Lymphomas describe white blood cell malignancies that:

Select one of the following:

  • do not involve bone marrow tissue initially

  • cause marked proliferation of white blood cells

  • initiate as cancers of lymphoid cells

  • all answers are correct

Explanation

Question 112 of 130

1

Malignant neoplasms involving lymphocyte proliferation in the lymph nodes are called:

Select one of the following:

  • lymphomas

  • leukaemias

  • myelomas

  • lymphocytomas

Explanation

Question 113 of 130

1

Which of the following substances acts as an anticoagulant?

Select one of the following:

  • Vitamin K

  • Prothrombin

  • Heparin

  • Fibrinogen

Explanation

Question 114 of 130

1

A 67-year-old female is admitted to the ED with a diagnosis of polycythaemia vera. This has occurred as a result of:

Select one of the following:

  • dehydration.

  • blood doping.

  • exposure to high altitude.

  • abnormal proliferation of red cells.

Explanation

Question 115 of 130

1

In disseminated intravascular coagulation (DIC), active bleeding occurs after intravascular clotting because:

Select one of the following:

  • clotting factors are depleted.

  • tissue factor (TF) is inactivated.

  • prothrombin is activated.

  • inflammatory mediators are released.

Explanation

Question 116 of 130

1

________ describes a condition where malignant white cells produce vast quantities of abnormal immunoglobulins that ultimately destroy bone.

Select one of the following:

  • Multiple myeloma

  • Chronic lymphocytic leukaemia

  • Agranulocytosis

  • Non-Hodgkin lymphoma

Explanation

Question 117 of 130

1

A neoplastic disorder that involves malignant plasma cells that replace the bone marrow and erodes bone is referred to as:

Select one of the following:

  • Aplastic anemia

  • Multiple myeloma

  • Leukaemia

  • Non-Hodgkin lymphoma

Explanation

Question 118 of 130

1

What are the typical early clinical manifestations of anaemia?

Select one of the following:

  • Jaundice, cyanosis

  • Pallor, dyspnea, and fatigue

  • Chest pain, palpitations

  • Bradycardia, cyanosis

Explanation

Question 119 of 130

1

Which of the following individuals is at highest risk for developing a vitamin B12 deficiency anaemia?

Select one of the following:

  • a 47-year-old male who had a gastrectomy procedure (removal of the stomach)

  • a 3-year-old female who is a fussy eater

  • a 64-year-old male with a history of duodenal ulcers and gastrointestinal bleeding

  • a 26-year-old female in the second trimester of her first pregnancy

Explanation

Question 120 of 130

1

A diverse group of neoplasms developing from the proliferation of malignant lymphocytes in the lymphoid system is referred to as:

Select one of the following:

  • Leukaemias

  • Microcytic anaemias

  • Lymphomas

  • Lymphatic anaemias

Explanation

Question 121 of 130

1

In which of the following individuals would a clinician most suspect multiple myeloma as a diagnosis?

Select one of the following:

  • A 70-year-old woman whose blood work reveals large numbers of immature granulocytes

  • An 81-year-old male resident of a long-term care home who has an uncommon bacterial pneumonia and who is unable to produce a fever

  • A 40-year-old man who has had three broken bones over the past 6 months and whose serum calcium and creatinine levels are elevated

  • A 68-year-old former coal miner who has white cell levels exponentially higher than normal ranges

Explanation

Question 122 of 130

1

A 34-year-old male presents in the ED with extreme fatigue and shortness of breath. His skin and sclera appear to have a yellowish discoloration. These findings are consistent with which type of anaemia?

Select one of the following:

  • pernicious anaemia

  • aplastic anaemia

  • iron deficiency anaemia

  • haemolytic anaemia

Explanation

Question 123 of 130

1

A 40-year-old female develops disseminated intravascular coagulation (DIC). The most likely cause of this condition is:

Select one of the following:

  • lack of vitamin B12.

  • blood transfusion.

  • snake venom.

  • sepsis.

Explanation

Question 124 of 130

1

A 52-year-old male IV drug user 5years ago was diagnosed with hepatitis C. He is now experiencing impaired blood clotting. A decrease in which of the following vitamins is most likely the cause?

Select one of the following:

  • E

  • K

  • B12

  • D

Explanation

Question 125 of 130

1

Chronic blood loss causes anaemia because of the:

Select one of the following:

  • lower metabolic rate

  • shortened life span of the erythrocytes

  • loss of protein and electrolytes

  • smaller amount of recycled iron available

Explanation

Question 126 of 130

1

What is the basic abnormality in thalassemia?

Select one of the following:

  • Several amino acids in the Haem chains have been replaced by substitute amino acids.

  • The iron molecule is displaced in haemoglobin.

  • More than four globin chains are found in the erythrocytes.

  • There is failure to synthesize either the alpha or beta chains in the haemoglobin molecule.

Explanation

Question 127 of 130

1

A 60-year-old woman is suspected of having non-Hodgkin lymphoma (NHL). Which of the following aspects of her condition would help to rule out Hodgkin lymphoma?

Select one of the following:

  • The woman complains of recent debilitating fatigue.

  • Her neoplasm originates in secondary lymphoid structures.

  • The lymph nodes involved are located in a large number of locations in the lymphatic system.

  • The presence of Reed-Sternberg cells has been confirmed.

Explanation

Question 128 of 130

1

One of the reasons non-Hodgkin’s lymphomas are harder to treat than Hodgkin’s lymphomas is that they

Select one of the following:

  • are not affected by the newer drug treatments.

  • are asymptomatic until they reach stage IV.

  • tend to be much larger than Hodgkin’s lymphomas.

  • involve multiple nodes and widespread metastases.

Explanation

Question 129 of 130

1

Individuals with liver disease often suffer from coagulation disorders because:

Select one of the following:

  • treatment medications for liver failure cause fibrinolysis.

  • the liver is often the site of platelet pooling.

  • high levels of bilirubin interfere with the clotting system.

  • clotting factors are produced in the liver.

Explanation

Question 130 of 130

1

In individuals with acute leukaemia, the increased number of malignant leukocytes leads to:
1. decreased haemoglobin.
2. thrombocytopenia.
3. bone pain only with increased activity.
4. splenomegaly.

Select one of the following:

  • 1,3

  • 1,2,4

  • 1,2,3,4

  • 2,3,4

Explanation